Difference between revisions of "2017 AMC 12B Problems/Problem 5"

(Created page with "The data set <math>[6, 19, 33, 33, 39, 41, 41, 43, 51, 57]</math> has median <math>Q2 = 40</math>, first quartile <math>Q1 = 33</math>, and third quartile <math>Q3 = 43</math>...")
 
Line 1: Line 1:
 +
==Problem 5==
 +
 
The data set <math>[6, 19, 33, 33, 39, 41, 41, 43, 51, 57]</math> has median <math>Q2 = 40</math>, first quartile <math>Q1 = 33</math>, and third quartile <math>Q3 = 43</math>. An outlier in a data set is a value that is more than <math>1.5</math> times the interquartile range below the first quartle (<math>Q1</math>) or more than <math>1.5</math> times the interquartile range above the third quartile (<math>Q3</math>), where the interquartile range is defined as <math>Q3 - Q1</math>. How many outliers does this data set have?
 
The data set <math>[6, 19, 33, 33, 39, 41, 41, 43, 51, 57]</math> has median <math>Q2 = 40</math>, first quartile <math>Q1 = 33</math>, and third quartile <math>Q3 = 43</math>. An outlier in a data set is a value that is more than <math>1.5</math> times the interquartile range below the first quartle (<math>Q1</math>) or more than <math>1.5</math> times the interquartile range above the third quartile (<math>Q3</math>), where the interquartile range is defined as <math>Q3 - Q1</math>. How many outliers does this data set have?
  
 
<math>\textbf{(A)}\ 0\qquad\textbf{(B)}\ 1\qquad\textbf{(C)}\ 2\qquad\textbf{(D)}\ 3\qquad\textbf{(E)}\ 4</math>
 
<math>\textbf{(A)}\ 0\qquad\textbf{(B)}\ 1\qquad\textbf{(C)}\ 2\qquad\textbf{(D)}\ 3\qquad\textbf{(E)}\ 4</math>
 +
 +
==Solution==
 +
 +
==See Also==
 +
{{AMC12 box|year=2017|ab=B|before=num-b=4|num-a=6}}
 +
{{MAA Notice}}

Revision as of 18:50, 16 February 2017

Problem 5

The data set $[6, 19, 33, 33, 39, 41, 41, 43, 51, 57]$ has median $Q2 = 40$, first quartile $Q1 = 33$, and third quartile $Q3 = 43$. An outlier in a data set is a value that is more than $1.5$ times the interquartile range below the first quartle ($Q1$) or more than $1.5$ times the interquartile range above the third quartile ($Q3$), where the interquartile range is defined as $Q3 - Q1$. How many outliers does this data set have?

$\textbf{(A)}\ 0\qquad\textbf{(B)}\ 1\qquad\textbf{(C)}\ 2\qquad\textbf{(D)}\ 3\qquad\textbf{(E)}\ 4$

Solution

See Also

2017 AMC 12B (ProblemsAnswer KeyResources)
Preceded by
num-b=4
Followed by
Problem 6
1 2 3 4 5 6 7 8 9 10 11 12 13 14 15 16 17 18 19 20 21 22 23 24 25
All AMC 12 Problems and Solutions

The problems on this page are copyrighted by the Mathematical Association of America's American Mathematics Competitions. AMC logo.png